LSAT and Law School Admissions Forum

Get expert LSAT preparation and law school admissions advice from PowerScore Test Preparation.

 Administrator
PowerScore Staff
  • PowerScore Staff
  • Posts: 8916
  • Joined: Feb 02, 2011
|
#26706
Please post below with any questions!
 mokkyukkyu
  • Posts: 97
  • Joined: Aug 17, 2016
|
#28273
Hi,
I was not sure between A and B...
I chose B in the end thinking it 's the combination of the 1st sentence and the second sentence.
Why is B wrong??
 Kbar100
  • Posts: 1
  • Joined: Sep 02, 2016
|
#28276
Here's why B is wrong.

The stimulus says that one needs to do the equivalent of 30 minutes of brisk walking on most days to get the cardiovascular benefits.

Answer choice B says that doing the equivalent of an hour of brisk walking two or three times a week generally produces dramatic improvements in cardiovascular health.

There's an equivocation error between "most days" in the stimulus and "Two or three times a week" in Answer choice B. A week has 7 days, and therefore most days means "4 or more days".

It's really hard to see it if you missed it when you went through the answers on your first pass. You have to nitpick every answer when you bring it down to two possible choices. When in doubt, compare the modality to the stimulus if you're doing MBT with modality.

NOTE: I just finished this preptest, timed, right now. I missed 1 in logic games (on the sundial game), 0 in both LR, and 3 in RC(-2 in passage 3, -1 in passage 4). I'm taking the LSAT in 3 weeks.

If you have any other questions on preptest 78 feel free to message me. There aren't many resources or explanations up for this prep test because it's fairly new, June 2016. And the best way for me to review LR is to explain the answers/why they're right and wrong.
 mokkyukkyu
  • Posts: 97
  • Joined: Aug 17, 2016
|
#28294
Hi

Thanks for your reply! Wow, that's fantastic...let me message you later then.
Would really appreciate your insight.
Thanks! :)
 Zierra28
  • Posts: 17
  • Joined: Aug 12, 2015
|
#28791
@Kbar, the way I broke it down is 1hour 2/3day is equal to .5hour 4-6days. Do you see my math? Was I TOO nitpicky?? :lol:

Help my mind resolve this please!
 Zierra28
  • Posts: 17
  • Joined: Aug 12, 2015
|
#28792
Ah! NEVERMIND!!! I see it "CAN produce" vs "Generally produces" AHHHHH those LSAT details ;)
 ChicaRosa
  • Posts: 111
  • Joined: Aug 23, 2016
|
#31349
I'm struggling to understand why A is correct?

I originally chose C but now I realized it's wrong because of the word "great" while in the stimulus it mentions "dramatic" Also, it's comparing that walking for half an hour is better than a strenuous workout while in the stimulus it doesn't do that. The stimulus mentions that a vigorous workout is more effective but it isn't necessary. Is my line of reasoning correct?

Is A correct because of combing the first and last sentence?

Thanks!
 Adam Tyson
PowerScore Staff
  • PowerScore Staff
  • Posts: 5153
  • Joined: Apr 14, 2011
|
#31355
A is the best answer, ChicaRosa, for the same reason that you rejected answer C - as you said, "The stimulus mentions that a vigorous workout is more effective." If walking gets dramatic results, and a vigorous workout is even better, then it must be true that a vigorous workout (which would certainly include a strenuous workout) can also produce dramatic results, right?

Take another look and see if that adds up for you. Thanks for asking!
 Etsevdos
  • Posts: 62
  • Joined: Oct 22, 2017
|
#41809
Adam Tyson wrote:A is the best answer, ChicaRosa, for the same reason that you rejected answer C - as you said, "The stimulus mentions that a vigorous workout is more effective." If walking gets dramatic results, and a vigorous workout is even better, then it must be true that a vigorous workout (which would certainly include a strenuous workout) can also produce dramatic results, right?

Take another look and see if that adds up for you. Thanks for asking!
Was between A/ C. Clearly A is better when reviewing untimed. I read Cs use of effective a bit differently. Effective I took as reacting quicker but ultimately obtaining the same result. Therefore, I guess I read C as potentially saying that Vig exc will obtain results faster (assuming over same period) but because it is not necessary, they would eventually even out. Nonetheless, I would agree that this is not strongly supported and A is better, but assuming A was not an answer choice, would this reasoning be invalid?

Also, generally on MBT I try to stick to weaker answers - B, D, E immediately became suspect. I focused on A / C - Is this correct approach, generally.
 Shannon Parker
PowerScore Staff
  • PowerScore Staff
  • Posts: 147
  • Joined: Jun 08, 2016
|
#42376
Etsevdos,

Yes it is a very helpful practice to quickly eliminate answers that are obviously wrong. For MBT questions this can easily be done by seeing if the answer "can be false."

As for your analysis of C, be careful with what you read into the stimulus and answer choices. The stimulus states that "vigorous exercise is more effective," but does not say anything about frequency of vigorous exercise. Therefore "more vigorous exercise is more effective" should be read as "more vigorous exercise will produce more significant cardiovascular health benefits."

Keep up the hard work

Get the most out of your LSAT Prep Plus subscription.

Analyze and track your performance with our Testing and Analytics Package.